using the following statements to compare how the aas congruence therom and the asa congruence therom are similar and how they are different.

Using The Following Statements To Compare How The Aas Congruence Therom And The Asa Congruence Therom

Answers

Answer 1

In summary, both theorems require two pairs of congruent angles, but the AAS Congruence Theorem requires an included side to be congruent while the ASA Congruence Theorem requires a non-included side to be congruent.

What is similarity theorem?

A similarity theorem is a statement in geometry that describes a relationship between similar geometric figures. Similar figures are figures that have the same shape but may have different sizes. A similarity theorem states that certain corresponding angles of similar figures are congruent and that the ratio of corresponding sides is constant. This constant ratio is called the scale factor, and it is used to find missing side lengths or to enlarge or reduce the size of a figure. The most commonly used similarity theorem is the AA (angle-angle) theorem, which states that if two angles of one triangle are congruent to two angles of another triangle, then the triangles are similar.

Here,

The AAS Congruence Theorem and the ASA Congruence Theorem are both used to prove that two triangles are congruent. However, they differ in the conditions required for the triangles to be congruent.

The AAS Congruence Theorem requires that two pairs of corresponding angles and one pair of corresponding included sides be congruent. This means that if two triangles have two pairs of congruent angles and a side included between them is congruent, then the triangles are congruent.

On the other hand, the ASA Congruence Theorem requires that two pairs of corresponding angles and one pair of corresponding non-included sides be congruent. This means that if two triangles have two pairs of congruent angles and a side not included between them is congruent, then the triangles are congruent.

To know more about similarity theorem,

https://brainly.com/question/24167205

#SPJ1


Related Questions

Determine the equation of the ellipse with foci (15,-2) and (3,-2), and co-vertices (9,6) and (9,-10).​

Answers

Answer: We know that the center of the ellipse is the midpoint of the line segment joining the foci. So, the center is ((15+3)/2, (-2-2)/2) = (9, -2).

The distance between the foci is 2c, where c is the distance between the center and each focus. So, 2c = 15 - 3 = 12, which means c = 6.

The distance between the center and each co-vertex is b. So, b = 10 - (-2) = 12.

The semi-major axis is a, which is the distance from the center to a vertex. Since we have the co-vertices, we can use the Pythagorean theorem to find a:

a^2 = b^2 - c^2

a^2 = 12^2 - 6^2

a^2 = 108

a = sqrt(108) = 6*sqrt(3)

Therefore, the equation of the ellipse is:

(x - 9)^2 / (6*sqrt(3))^2 + (y + 2)^2 / 12^2 = 1

Simplifying, we get:

(x - 9)^2 / 72 + (y + 2)^2 / 144 = 1

So, the equation of the ellipse is (x - 9)^2 / 72 + (y + 2)^2 / 144 = 1.

Enjoy!!!!!!!!!!!!!!!!!!!!!!

Question 6 of 10
Ashlee is purchasing a $125,000 home and her bank is offering her a 30-year
mortgage at a 4.75% interest rate. In order to lower her monthly payment,
Ashlee will make a 20% down payment and is considering a purchase of 2
points. How much lower will her monthly payment be if she purchases the
points?
A. $16.69
B. $15.98
C. $12.09
D. $14.96
SUBMIT

Answers

Answer:

D

Step-by-step explanation:

First, let's calculate the initial monthly payment without purchasing any points.

The amount of the mortgage after the 20% down payment is:

$125,000 x 0.8 = $100,000

To calculate the monthly payment, we can use the formula:

M = P [ i(1 + i)^n ] / [ (1 + i)^n – 1]

Where:

M = Monthly payment

P = Principal (loan amount)

i = Interest rate (per month) = 4.75% / 12 = 0.0039583

n = Number of payments = 30 years x 12 months per year = 360

So, plugging in the values:

M = $100,000 [ 0.0039583(1 + 0.0039583)^360 ] / [ (1 + 0.0039583)^360 – 1]

M = $522.12 (rounded to the nearest cent)

Now, let's calculate the new monthly payment if Ashlee purchases 2 points. Each point costs 1% of the loan amount, so for a $100,000 loan, each point is $1,000.

By purchasing 2 points, Ashlee would pay an additional $2,000 upfront (2 x $1,000). This will lower the interest rate on the loan by 0.5%, from 4.75% to 4.25%.

Using the same formula as before, but with the new interest rate of 4.25% / 12 = 0.0035417, we get:

M = $100,000 [ 0.0035417(1 + 0.0035417)^360 ] / [ (1 + 0.0035417)^360 – 1]

M = $505.16 (rounded to the nearest cent)

The difference in monthly payments is:

$522.12 - $505.16 = $16.96

Rounding to the nearest cent, the answer is option D, $14.96.

Determine the interval(s) over which the graph of f left parenthesis x right parenthesis equals short dash x to the power of 6 minus 6 x to the power of 5 plus 12 x minus 2 is concave up or concave down.

Answers

Answer:

gdiduusuehfuehvv

Step-by-step explanation:

bbzbhshxhdhhd are the best and my mom I WINNNNNN you want it to be over with you guys to come in and your mom are doing ghhdhhsuwuwhsh and your family are doing well and your family are doing well and your family are doing well and your family are doing well and your family and my wife was the best and I WINNNNNN and my mom are doing ghhdhhsuwuwhsh and what is the only ones I WINNNNNN with the kids I WINNNNNN and I will be over and I can do i and eat early for you and Darshanie and I WINNNNNN and my friend and your mom and my mom ever had a great day I love ever and your mom are the kids I will get you a great mom and I will send you a link and I WINNNNNN with the only ones I WINNNNNN with you don't need me I can get it and I will be over and your mom and your mom I will send it out tomorrow night I will send it out tomorrow night with you guys and I can send the best ones that have to do i I will send you and Darshanie and your mom I will send it and your family and I WINNNNNN and my friend are doing errands to run out to do i you guys and I WINNNNNN you want me I will send you and Darshanie is done now we have a great day for a great mom I will be over there and I WINNNNNN you and everyone and my friend and my friend and what time do the kids have a whole new world we live together I will be home tomorrow night with the only one that has to wait until the way home tomorrow morning to run out of wo you guys to run out with the best ones that w with the kids I will get you guys want me when we good day for you it

Use the Law of Cosines to solve the problem. You must solve for BC first. Solve this order

A ship travels due west for 94 miles. It then travels in a northwest direction for 119 miles and ends up 173 miles from its original position. To the nearest tenth of a degree, how many degrees north of west (x) did it turn when it changed direction? Show you Work.

Answers

answer: 175

Step-by-step explanation:

Answer:

71.9 degrees

Step-by-step explanation:

The find the value of x on the given diagram, we must first find the included angle between the two line segments labelled 94 mi and 119 mi in the triangle. To do this, we can use the Law of Cosines.

[tex]\boxed{\begin{minipage}{6 cm}\underline{Law of Cosines} \\\\$c^2=a^2+b^2-2ab \cos C$\\\\where:\\ \phantom{ww}$\bullet$ $a, b$ and $c$ are the sides.\\ \phantom{ww}$\bullet$ $C$ is the angle opposite side $c$. \\\end{minipage}}[/tex]

The values to substitute into the formula are:

a = 119b = 94c = 173

Solving for the angle C:

[tex]\begin{aligned}173^2&=119^2+94^2-2(119)(94)\cos C\\\\29929&=14161+8836-22372\cos C\\\\29929&=22997-22372\cos C\\\\6932&=-22372\cos C\\\\-\dfrac{6932}{22372}&=\cos C\\\\C&=\cos^{-1}\left(-\dfrac{6932}{22372}\right)\\\\C&=108.0502874...^{\circ}\end{aligned}[/tex]

As angles on a straight line sum to 180°, the value of x can be calculated by subtracting the found value of C from 180°:

[tex]x=180^{\circ}-108.0502874...^{\circ}[/tex]

[tex]x=71.9497125...^{\circ}[/tex]

[tex]x=71.9^{\circ}\; \sf (nearest\;tenth)[/tex]

Therefore, the ship turned 71.9 degrees north of west when it changed direction.

At age ​39, you start saving for retirement. If your investment plan pays an APR of ​4% and you want to have ​$0.8 million when you retire in 26 ​years, how much should you deposit​ monthly?

Answers

Answer:

Step-by-step explanation:

20/8as a mixed number in its simplest form

Answers

Answer:

2 1/2

Step-by-step explanation:

20/8=2r4

2r4= 2 4/8

2 4/8 = 2 1/2

Answer:

2 1/2

Step-by-step explanation:

Step 1:

We first want to find the whole number, and to do this we divide the numerator by the denominator. Since we are only interested in whole numbers, we ignore any numbers to the right of the decimal point.

20/8= 2.5 = 2

Now that we have our whole number for the mixed fraction, we need to find our new numerator for the fraction part of the mixed number.

Step 2: Get the new numerator

To work this out we'll use the whole number we calculated in step one (2) and multiply it by the original denominator (8). The result of that multiplication is then subtracted from the original numerator:

20 - (8 x 2) = 4

Step 3: Our mixed fraction

We've now simplified 20/8 to a mixed number. To see it, we just need to put the whole number together with our new numerator and original denominator:

2 4/8

Step four: Convert it by simplifying

2 4/8 = 2 1/2

Question
Jaq rode their bike for 3 1/3
hours at 15 miles per hour. How far did they ride?

Answers

Answer:

Step-by-step explanation:

We can use the formula distance = rate x time to solve the problem.

The rate is 15 miles per hour and the time is 3 1/3 hours.

To work with the time, we need to convert it to an improper fraction:

3 1/3 = (3 x 3 + 1)/3 = 10/3

Now we can plug in the values:

distance = 15 x 10/3

distance = 50 miles (rounded to the nearest mile)

Therefore, Jaq rode 50 miles.

Solve for x. Pls help

Answers

value of variable x in the triangle is 8.

Define Triangle Proportionality Theorem

The Triangle Proportionality Theorem, also known as the Side-Splitter Theorem, states that if a line is parallel to one side of a triangle and intersects the other two sides, then it divides those sides proportionally. In other words, if a line is drawn parallel to one side of a triangle, and intersects the other two sides, then the ratio of the lengths of the line segments it creates on those sides is equal.

More formally, let ABC be a triangle, and let D be a point on the line segment BC that is between B and C. If the line through D parallel to AB intersects AC at point E, then BD/DC = AE/EC. This theorem is useful in many geometrical proofs and can be used to solve problems involving similar triangles.

According to Triangle Proportionality Theorem

4x-7/5=20/4

4x-7=5×5

4x=32

x=8

Hence, value of variable x in the triangle is 8.

To know more about line, visit:

https://brainly.com/question/30003330

#SPJ1

I really need help and there also is a part c and d

Answers

Part A: The probability of rolling a 5 is 1/6 or approximately 0.167. Part B: the probability of rolling an even number is 3/6 or 1/2 or 0.5.

Describe probability ?

Probability is a branch of mathematics concerned with measuring the likelihood or chance of an event occurring. It is defined as the ratio of the number of favorable outcomes to the total number of possible outcomes. Probability is expressed as a number between 0 and 1, where 0 means that the event will not occur and 1 means that the event will definitely occur. For example, if the probability of an event is 0.5, it means that the event has an equal chance of occurring or not occurring. Probability is used in various fields, such as science, engineering, finance, and statistics, to make predictions and make decisions based on uncertain events.

Part A:

The number cube has six faces, and each face has an equal chance of landing face-up. Therefore, the probability of rolling a 5 is 1/6 or approximately 0.167.

Part B:

The even numbers on a number cube are 2, 4, and 6. There are three even numbers out of a total of six possible outcomes. Therefore, the probability of rolling an even number is 3/6 or 1/2 or 0.5.

To know more about outcomes visit:

https://brainly.com/question/30919037

#SPJ1

Jason invested $5,500 in an account paying an interest rate of 1 7/8 ​ % compounded quarterly. Kayden invested $5,500 in an account paying an interest rate of 1 3/8 ​ % compounded annually. After 8 years, how much more money would Jason have in his account than Kayden, to the nearest dollar?

Answers

Using compounding we know that the additional amount Jason has more than Kayden is $249.48.

What is compounding?

Calculating interest on the principal borrowed as well as any prior interest.

In order to compute compound interest, multiply the principle of the original loan by the annual interest rate multiplied by the number of compound periods minus one.

So, the amount of Jason after 8 years:

Amount: $5500

Interest: 1.875%

Compounded: Quarterly

Using a compounding calculator:

Amount after 8 years: $6,387.85

The amount of Kayden:

Amount: $5500

Interest: 1.375%

Compounded: Quarterly

Using a compounding calculator:

Amount after 8 years: $6,138.37

The additional amount Jason got: 6,387.85 - 6,138.37 = $249.48


Therefore, using compounding we know that the additional amount Jason has more than Kayden is $249.48.

Know more about compounding here:

https://brainly.com/question/28020457

#SPJ1

Answer:253

Step-by-step explanation:

Please help me answer part 1, part 2, and part 3 question ASAP!!
Will mark as brainliest if correct and 50+ points!

Answers

Answer:

Step-by-step explanation:

Section 1

1: 2x + 20

2. 8x - 49
3: 7x + 21

4.8x - 4

5. 15x + 5

6. 12x + 15

7. 20x - 50

8. 21x + 14

9. 24x - 6

10. 45x - 18

11. 18x - 8

12. 30x + 9

13. 24x + 36

14. 25x - 20

15. 14x - 63

16. 32x + 20

17. 24x - 10

18. 42x - 18

19. 24x - 80

20. 48x - 32

please help me with math quiz i’ll give you brainlist

Answers

The correct answer is Skewed

Answer:

Answer: B. Symmetric.

Explanation:

In a symmetric distribution, the data is evenly distributed around the mean or median, creating a mirror image on both sides of the center. In this histogram, the median and mean are very close together at 55 and the bars on both sides of the center are roughly equal in height, indicating a fairly even distribution. Therefore, the histogram is symmetric.

a circle has a radius of 10 cm, find the perimeter

Answers

Answer:

The perimeter of a circle is also known as the circumference. The formula for the circumference of a circle is:

C = 2πr

where C is the circumference, π is a constant approximately equal to 3.14, and r is the radius of the circle.

Substituting the given value, we get:

C = 2 x 3.14 x 10

C = 62.8 cm

Therefore, the perimeter of the circle is 62.8 cm.

C = 2 x 3.14 × 10
C = 62.8 cm

Assume that the readings at freezing on a batch of thermometers are normally distributed with a mean of 0°C and a standard deviation of 1.00°C. A single thermometer is randomly selected and tested. Find the probability of obtaining a reading between 0°C and 1.08°C. Round your answer to 4 decimal places

Answers

Answer: We are given that the readings at freezing on a batch of thermometers are normally distributed with a mean of 0°C and a standard deviation of 1.00°C.

To find the probability of obtaining a reading between 0°C and 1.08°C, we need to calculate the z-scores for these values using the formula:

z = (x - mu) / sigma

where x is the value we are interested in, mu is the mean, and sigma is the standard deviation.

For x = 0°C, we have:

z1 = (0 - 0) / 1.00 = 0

For x = 1.08°C, we have:

z2 = (1.08 - 0) / 1.00 = 1.08

Using a standard normal table or a calculator, we can find the probability of obtaining a z-score between 0 and 1.08.

Using a standard normal table or a calculator, we find that the probability of obtaining a z-score between 0 and 1.08 is 0.3583.

Therefore, the probability of obtaining a reading between 0°C and 1.08°C is 0.3583, rounded to 4 decimal places.

Step-by-step explanation:

given two nonzero vectors which aren't parallel, there is exactly one plane parallel to both vectors. g

Answers

The argument  using the components of the non zero vectors which are not parallel are as follow, sa1 + tb1 = c1

sa2 + tb2 = c2

sa3 + tb3 = c3

Geometric argument,

a and b are nonzero vectors that are not parallel.

A plane in three-dimensional space.

Any vector in this plane can be written as a linear combination of a and b.

Let c be any vector in this plane.

Construct a parallelogram with sides a and b, and the diagonal of the parallelogram passing through the point c.

This diagonal is the vector c written as a linear combination of a and b, say c = sa + tb for some scalars s and t.

Diagonal of a parallelogram bisects and is bisected by its opposite sides.

c lies on the line passing through the midpoints of a and b.

c can be expressed as a linear combination of a and b.

Argument using components,

a and b are nonzero vectors that are not parallel, they are linearly independent.

Any vector in the plane determined by a and b can be expressed as a linear combination of a and b.

Let c = (c1, c2, c3) be any vector in this plane.

Now, c as c = sa + tb, where s and t are scalars to be determined.

Writing a and b in terms of their components, we have,

a = (a1, a2, a3) and b = (b1, b2, b3)

Then, we can write c as,

c = (s a1 + t b1, s a2 + t b2, s a3 + t b3)

Values of s and t such that c has the components (c1, c2, c3).

Equating the components of c with those of (c1, c2, c3), to get the system of equations,

sa1 + tb1 = c1

sa2 + tb2 = c2

sa3 + tb3 = c3

Therefore, solution of the argument of non zero vectors solved using standard techniques such as Gaussian elimination or Cramer's rule.

Learn more about vectors here

brainly.com/question/31058167

#SPJ4

The above question is incomplete, I answer the question in general according to my knowledge:

Suppose that a and b are nonzero vectors that are not parallel and c is any vector in the plane determined by a and b. Give a geometric argument to show that c can be written as c=sa +tb for suitable scalars s and t. Then give an argument using components.

one ticket is drawn at random from each of the two boxes below: 1 2 6 1 4 5 8 find the chance that the both numbers are even numbers.

Answers

The chance that both numbers drawn are even numbers is 8/21.

The probability refers to the measure of the likelihood or chance of an event occurring. It is a numerical value between 0 and 1, where 0 indicates that the event is impossible, and 1 indicates that the event is certain.

There are 4 even numbers and 3 odd numbers in the first box, and 2 even numbers and 1 odd number in the second box.

The probability of drawing an even number from the first box is 4/7, and the probability of drawing an even number from the second box is 2/3.

By the multiplication rule of probability, the probability of drawing an even number from both boxes is

(4/7) × (2/3) = 8/21

Learn more about probability here

brainly.com/question/11234923

#SPJ4

Between which two consecutive integers does [tex]\sqrt138[/tex]lie?

Answers

The square root of 138 lies between 11 and 12, as 11²=121 and 12²=144.

What is number?

Number is a mathematical object used to count, measure, and label. Numbers are used in almost every field of mathematics and science, including algebra, calculus, geometry, physics, and computer science. Numbers can also be used to represent data, such as population, income, temperature, or time. In addition, numbers are used to represent abstract concepts, such as love, truth, beauty, and justice.

This is because the square root of a number is the number that, when multiplied by itself, produces the original number. Therefore, to find the square root of 138, we need to identify two consecutive integers such that one of them squared is smaller than 138 and the other squared is larger than 138.

To do this, we can work our way up from the integer closer to 0, in this case 11. 11 squared is 121, which is smaller than 138, so we know that the square root of 138 must be between 11 and a larger integer. Then, if we square 12, we get 144, which is larger than 138. Therefore, we can definitively say that the square root of 138 lies between 11 and 12.

To know more about number click-
https://brainly.com/question/24644930
#SPJ1

The square root of 138 lies between 11 and 12, as 11² is 121 and 12² is 144.

What is number?

Number is a mathematical object used to count, measure, and label. Numbers are used in almost every field of mathematics and science, including algebra, calculus, geometry, physics, and computer science. Numbers can also be used to represent data, such as population, income, temperature, or time. In addition, numbers are used to represent abstract concepts, such as love, truth, beauty, and justice.

To calculate this, we can divide 138 by 11 and 12, and see which integer is closer to the answer.

138 divided by 11 is 12.545454545454545454545454545455.
138 divided by 12 is 11.5.

Since 11.5 is closer to the answer, the square root of 138 lies between 11 and 12.

To know more about number click-
brainly.com/question/24644930
#SPJ1

you are computing a confidence interval for the difference in 2 population proportions. which of the following could be negative? select all.OP1Op 1 - 2Standard errorCritical valueLower bound of the confidence intervalUpper bound of the confidence interval

Answers

For the computation of confidence interval for the difference in two population proportions following are negative,

p₁(cap) - p₂(cap)

Lower bound of the confidence interval

Upper bound of the confidence interval

For the computation of confidence interval,

The difference in two population proportions,

p₁ - p₂, can be negative or positive.

This implies,

The sample estimate of the difference in proportions,

p₁(cap) - p₂(cap), can also be negative or positive.

The standard error and critical value are always positive values and cannot be negative.

The lower and upper bounds of the confidence interval can be negative or positive.

Depending on the sample estimate and the margin of error.

So, both the lower and upper bounds can be negative.

Learn more about confidence interval here

brainly.com/question/15049116

#SPJ4

The above question is incomplete, the complete question is:

You are computing a confidence interval for the difference in 2 population proportions. which of the following could be negative?

Select all.

a. p₁

b. p₁(cap) - p₂(cap)

c. Standard error

d. Critical value

e. Lower bound of the confidence interval

f. Upper bound of the confidence interval

1. If the angle between the vectors a and b is π/4 and | a × b | = 1, then a. b is equal to

Answers

Answer:

We can use the formula |a × b| = |a| |b| sin θ to solve for the magnitude of the cross product |a × b|, where θ is the angle between vectors a and b. In this case, we have |a × b| = 1 and θ = π/4, so we can write:

1 = |a| |b| sin(π/4)

Simplifying, we have:

|a| |b| = √2

Now, we need to find the dot product a · b. We know that:

a · b = |a| |b| cos θ

where θ is the angle between vectors a and b. Since we're given the angle between a and b, we can substitute θ = π/4 and use the value we found for |a| |b|:

a · b = (√2) cos(π/4) = (√2)/2

Therefore, a · b is equal to (√2)/2.

Step-by-step explanation:

A baseball diamond is shaped like a square. The perimeter is 360 feet. What is the length of each side?

Answers

Answer: 90 feet

Step-by-step explanation:

Since the baseball diamond is shaped like a square, all sides are equal in length.

Let's denote the length of each side by x.

The perimeter of a square is the sum of the lengths of all four sides, so we can write:

4x = 360

Dividing both sides by 4, we get:

x = 90

Therefore, each side of the baseball diamond is 90 feet long.

Next > Pretest: Essential Learning 13 Select the correct answer. What is the simplified form of this expression? (5x² + 2x + 11) − (7 + 4x - 2x²) ​

Answers

Answer:

7x² - 2x + 4

Explanation:

(5x² + 2x + 11) − (7 + 4x - 2x²) ​

= 5x² + 2x + 11 - 7 - 4x + 2x²

= 7x² + 2x + 11 - 7 - 4x

= 7x² - 2x + 11 - 7

= 7x² - 2x + 4

So, the answer is 7x² - 2x + 4

Let f(X)=x-8 and g(X) =4x^2. Perform the function operation and then find the domain of the result. (f-g)(x)

Answers

Answer:

(f - g)(x) = -4x² + x - 8

Step-by-step explanation:

(f - g)(x) = f(x) - g(x)

            = x - 8 - (4x²)

            = -4x² + x - 8

Divide 18a6b2 by 9a3b.

2a9b3
2a2b2
2a3b2
2a3b

Answers

Answer:  Choice D)  [tex]2a^3b[/tex]

Work Shown:

[tex]\frac{18a^6b^2}{9a^3b}=\frac{18a^6b^2}{9a^3b}\\\\\frac{18a^6b^2}{9a^3b}=\frac{18}{9}*\frac{a^6}{a^3}*\frac{b^2}{b}\\\\\frac{18a^6b^2}{9a^3b}=2a^{6-3}b^{2-1}\\\\\frac{18a^6b^2}{9a^3b}=2a^3b\\\\[/tex]

The formula used on the 3rd line was (a^b)/(a^c) = a^(b-c). We subtract the exponents when dividing exponential expressions with the same base.

Gladtown School hosted Game Night
for students and their families. A total
of 240 people attended.
There were 16 tables set up in the school
cafeteria for the event. Every table had
the same number of people. How many
people sat at each table?

Answers

Answer:

15 people at each table

Step-by-step explanation:

1) divide total number of people by number of tables

240/16

2) solve

15 people at each table

Tell me which brand or which size is a better buy.

Answers

Answer:

The answer is brand B

Step-by-step explanation:

You divide $14.88 by 24 which equals 68 cents per item.

Then brand B is 60 cents per item which is the better buy!

The sun of two numbers is $189 and the difference is 15. What are the two numbers?

Answers

Answer:

102 and 87

Step-by-step explanation:

Let the two numbers be x and y.

x+y = 189

x-y =15

Add the two equations together.

x+y = 189

x-y =15

-------------------

2x =204

2x/2 = 204/2

x = 102

Now we can find y.

x+y = 189

102+y =189

y = 189-102

y =87

PLS HELP ME with all 4 questions!!

Answers

By answering the presented question, we may conclude that so by SAS congruency property we have said that all these triangles are similar.

What precisely is a triangle?

A triangle is a closed, a double geometric shape made up of three line segments known as sides that connect at three parameters known as vertices.

Triangles are differentiated by their angles and their sides. Triangles can be collinear (all sides equal), angles, or scalene dependent on their sides.

Triangles are classed as acute (all angles just under 90 degrees), right (one angle of approximately to 90 degrees), or ambiguous (all angles greater than 90 degrees).

The area of a triangle may be determined with the formula A = (1/2)bh, where A is the surface, b is really the right triangle base, and h is the triangle's height

here, from the figure, we have

Two sides of the triangle are equal.

And also all angles are also same.

Therefore, so by SAS congruency property we have say that all these triangles are similar.

Learn more about triangle here:

https://brainly.com/question/2773823

#SPJ1

Please help !!!! Given m∥n, find the value of x.

(3x+5) (x-25) = 180 degrees

Answers

Answer:

Step-by-step explanation:

[tex](3x+5)+(x-25)=180 \text{ \ (angles on a straight line are supplementary)}[/tex]

                 [tex]4x-20=180[/tex]

                         [tex]4x=200[/tex]         (+20 both sides)

                           [tex]x=50[/tex]           (÷4 both sides)

3x+5+x-25
4x-20=180
4x-20+20=180+20
4x=200
Divide both sides by the same factor

Cancel terms that are in both the numerator and denominator
Divide the numbers
4
x
4
=
200
4
4

4
=
2
0
0
4
x
=
50

=
5
0

​Zeros: −9​, multiplicity​ 1; −1​, multiplicity​ 2; degree 3

Form a polynomial whose zeros and degree are given.

Answers

Answer:

Step-by-step explanation:

If the zeros and their multiplicities are given, we can write the polynomial as the product of linear factors corresponding to each zero.

For this problem, the polynomial has zeros of -9 (multiplicity 1) and -1 (multiplicity 2), so the linear factors are:

(x + 9) and (x + 1)^2

To find the third factor, we use the fact that the degree of the polynomial is 3. We can multiply the linear factors together and then simplify:

(x + 9)(x + 1)^2 = (x^2 + 10x + 9)(x + 1)

= x^3 + 11x^2 + 19x + 9

Therefore, the polynomial with zeros of -9 (multiplicity 1), -1 (multiplicity 2), and degree 3 is:

f(x) = x^3 + 11x^2 + 19x + 9

when a certain number is doubled and then decreased by 9, the result is not more than 19. Find the range of values of the number​

Answers

Answer:

The number must be at most 10.

If the number is doubled and then decreased by 9, the result is 2x - 9.

2x - 9 ≤ 19

2x ≤ 28

x ≤ 14

Therefore, the range of values of the number is 0 to 10.

Other Questions
A flat portion of an average total cost curve represents the various different levels of output at which the firm achieves:a. decreasing returns to scaleb. diseconomics of scalec. economics of scaled. constant returns to scale In a class of students, the following data table summarizes how many students passeda test and complete the homework due the day of the test. What is the probability thata student completed the homework given that they failed the test?Completed the homeworkDid not complete the homeworkPassed the test Failed the test823 Stacy, Eileen, and Michelle are on a basketball team. Stacy scored 15% of the points, Eileenscored 10% and Michelle scored 13% of their team's points. If their team had a totalpoints, how many points did the remaining players on the team score? A(n) _____ is a set of integrated programs that manages the vital business operations for an entire multi-site, global organization. when a driver encounters a strong gust of wind, the driver should 15. edra wants to compare 2021 sales totals to the sales totals for 2020 and needs to add the 2020 data to the consolidated sales worksheet. open the file support ex19 5a 2020 sales.xlsx. switch back to the np ex19 5a firstlastname 2.xlsx workbook and go to the consolidated sales worksheet. create external references as follows: a. link cell g6 in the consolidated sales worksheet to cell f6 in the consolidated sales 2020 worksheet in the support ex19 5a 2020 sales.xlsx workbook. b. link cell g7 in the consolidated sales worksheet to cell f7 in the consolidated sales 2020 worksheet in the support ex19 5a 2020 sales.xlsx workbook. c. link cell g8 in the consolidated sales worksheet to cell f8 in the consolidated sales 2020 worksheet in the support ex19 5a 2020 sales.xlsx workbook. d. link cell g9 in the consolidated sales worksheet to cell f9 in the consolidated sales 2020 worksheet in the support ex19 5a 2020 sales.xlsx workbook. e. do not break the links. close the support ex19 5a 2020 sales.xlsx workbook. When a person is nearsighted in one eye and farsighted in the other this condition is referred to as? Conduct a survey with a minimum of 20 people.Complete the designed questionnaire in 1.2.Remind participants why your doing the survey and that their information will kept confidential If four molecules of carbon dioxide enter the Calvin cycle (four "turns" of the cycle), how many G3P molecules are produced and how many are exported? a. 4 G3P made, 1 G3P exported b. 4 G3P made, 2 G3P exported c. 8 G3P made, 1 G3P exported d. 8 G3P made, 4 G3P exported Life Orientation source based task one 2023 Using the set of data below, which statements are true? 13, 11, 16, 12, 42, 8The mean of this data set is 17.The median is 12.5.The median is 14.5.The mean is more affected by the outlier.The median is more affected by the outlier.The mean of this data set is 12. In an aquarium, the ratio of dolphins to pufferfish is 2: 3 and the ratio of pufferfish to starfish is 4: 7. There are 8 dolphins in the aquarium. How many starfish are there? Choose the letter of the item that best completes the statement or answers the question.1. According to the chapter, (a) a budget is a plan for managing money (b) not every family needs a budget (c) budgets are very complicated (d) everyone could use the same standard budget.2. Budgets are valuable for (a) rich families only (b) middle-income families only (c) poor families only (d) all families.3. As a first step in financial planning, one should (a) define those personal goals that will cost money (b) borrow enough money to pay all the expenses you are likely to incur (c) track your income and expenses for one month and then use the results to prepare a budget (d) shop wisely and use credit wisely.4. Budgets should be (a) followed very strictly (b) flexible (c) unchang- ing (d) complicated.5. Putting first things first in a budget means that (a) your needs and wants are equally important (b) whatever comes to your mind first is usually the most important thing (c) even the smallest expenses are included (d) you must first plan obtaining money and spending for those things that are most important to you.a typical students budget during the school year? (a) saving for a car (b) snacks (c) dating expenses (d) car fare to school.7. Which one of the following items is part of estimated income? (a) savings (b) school supplies (c) earnings (d) lunches.8. Wise money management (a) helps you to make the most of the money that you have (b) reduces the need for bank accounts (c) makes saving unnecessary (d) results in an increase in your al- lowance or wages.9. The price tag on both the West Coast and the Hippity Hop CDs was $14. Although Lonnie would have loved to buy both, they were too expensive. After much thought, Lonnie purchased the Hippity Hop recording. What was the opportunity cost of Lonnies decision? (a) $14 (b) the West Coast CD (c) $28 (d) the Hippity Hop CD When insiders have a much greater impact on the wage bargaining process than do outsiders, the negotiated wage is likely to be _____ the equilibrium wage.a) much greater than,b) much less than,c) almost equal to,d) about one-half of black and white breakfast cereal is called ?new york times crossword cdnas aid in identifying genes in eukaryotic genomes because Write the formula for the conjugate acid of each of the following bases.Express your answer as a chemical formula.a)C2H5NH2b)ClO4-c)HPO42-d)HCO3- approach in language teaching? Explain the advantages ofcommunicative approach in the field of ELT. Solve for the force in members GF, CD and FC and state whether it is in tension or compression using the method of sections. The horizontal member length is L = 20 ft. Take P=6 kip. P H P 2P/3 T IG 0.8L P/2 0.4L E B C ID - L - L - L - Hulme Company operates a small manufacturing facility as a supplement to its regular service activities. At the beginning of the current year, an asset account for the company showed the following balances:Manufacturing equipment$ 120,000Accumulated depreciation through the end of last year57,600During the current year, the following expenditures were incurred for the equipment:Major overhaul of the equipment on January 2 the current year that improved efficiency$ 13,000Routine repairs on the equipment1,000The equipment is being depreciated on a straight-line basis over an estimated life of 15 years with a $12,000 estimated residual value. The annual accounting period ends on December 31.PART 1:Prepare the adjusting entry that was made at the end of last year for depreciation on the manufacturing equipment. Note: If no entry is required for a transaction/event, select "No journal entry required" in the first account field. PART 2:2. Starting at the beginning of the current year, what is the remaining estimated life? What is the remaining life in years?PART 3:. Prepare the journal entries to record the two expenditures during the current year. Note: If no entry is required for a transaction/event, select "No journal entry required" in the first account field.